Laplace transform heaviside function

Click For Summary
The discussion revolves around finding the inverse Laplace transform of (e^(-s))/(s^2 + pi^2). The initial approach led to an incorrect result of u(t-1)cos(pi(t-1)), while the correct answer involves (sin(pi(t-1)/pi)u(t-1). Participants clarify that the presence of the step function arises from the convolution of u(t-1) and cos(pi*t). A method is suggested for handling transforms with e^(-as) by inverting F(s) and applying translation and truncation, which simplifies the process and avoids convolution. The conversation emphasizes the importance of understanding the distinction between multiplication and convolution in these transformations.
vvl92
Messages
13
Reaction score
0
I have a question asking for the inverse laplace transform of (e^(-s))/(s^2+pi^2).
I split it up to (e^(-s))/s x s/(s^2+pi^2) and got u(t-1)cos(pi(t-1)),but the correct answer is (sin(pi(t-1)/pi)u(t-1). So here it was split up to (e^(-s))/pi x pi/(s^2+pi^2) and I don't understand where the step function came from. It is the same in all of the questions and on things I have seen online, even though the transform of U(t-a) is (e^(-as))/s.
 
Physics news on Phys.org
vvl92 said:
I have a question asking for the inverse laplace transform of (e^(-s))/(s^2+pi^2).
I split it up to (e^(-s))/s x s/(s^2+pi^2) and got u(t-1)cos(pi(t-1)),but the correct answer is (sin(pi(t-1)/pi)u(t-1). So here it was split up to (e^(-s))/pi x pi/(s^2+pi^2) and I don't understand where the step function came from. It is the same in all of the questions and on things I have seen online, even though the transform of U(t-a) is (e^(-as))/s.

The inverse laplace transform of exp(-s)/s * s/(s^2 + pi^2) is the convolution of u(t-1) and cos(pi*t); that is, it equals
\int_0^t u(\tau - 1) \cos(\pi (t - \tau))\, d \tau.
 
When trying to inverse transforms that have a factor of ##e^{-as}## I like to use the following idea. Consider the direct transform$$
\mathcal Lf(t-a)u(t-a) = \int_0^\infty e^{-st}f(t-a)u(t-a)\, dt =
\int_a^\infty e^{-st}f(t-a)\, dt $$Now let ##w = t-a## $$
=\int_0^\infty e^{-s(w+a)}f(w)\, dw= e^{-as}F(s)$$
where ##F(s)## is the transform of ##f(t)##. What this says in terms of inversing is that if you have a form ##e^{-as}F(s)## to invert, you can just invert the ##F(s)## to ##f(t)##, then translate and truncate your answer to ##f(t-a)u(t-a)##. This avoids doing the convolution. Try it on your problem.
 
LCKurtz said:
When trying to inverse transforms that have a factor of ##e^{-as}## I like to use the following idea. Consider the direct transform$$
\mathcal Lf(t-a)u(t-a) = \int_0^\infty e^{-st}f(t-a)u(t-a)\, dt =
\int_a^\infty e^{-st}f(t-a)\, dt $$Now let ##w = t-a## $$
=\int_0^\infty e^{-s(w+a)}f(w)\, dw= e^{-as}F(s)$$
where ##F(s)## is the transform of ##f(t)##. What this says in terms of inversing is that if you have a form ##e^{-as}F(s)## to invert, you can just invert the ##F(s)## to ##f(t)##, then translate and truncate your answer to ##f(t-a)u(t-a)##. This avoids doing the convolution. Try it on your problem.

I agree. This is the way I would have chosen to do the problem, but he did ask a direct question, and I felt it important to answer, if only to dispel his misunderstanding about multiplication vs. convolution..
 
Question: A clock's minute hand has length 4 and its hour hand has length 3. What is the distance between the tips at the moment when it is increasing most rapidly?(Putnam Exam Question) Answer: Making assumption that both the hands moves at constant angular velocities, the answer is ## \sqrt{7} .## But don't you think this assumption is somewhat doubtful and wrong?

Similar threads

  • · Replies 1 ·
Replies
1
Views
965
  • · Replies 1 ·
Replies
1
Views
1K
  • · Replies 3 ·
Replies
3
Views
3K
  • · Replies 10 ·
Replies
10
Views
2K
Replies
2
Views
2K
Replies
1
Views
2K
  • · Replies 4 ·
Replies
4
Views
3K
Replies
3
Views
2K
  • · Replies 31 ·
2
Replies
31
Views
4K
Replies
3
Views
2K